Tutors’ Report on the EQE 2022 Papers
and the Meeting between Tutors and EQE Committees

N. Cordes (NL), L. Ferreira (PT), A. Valborg Guðmundsdóttir (IS), A. Hards (DE), H. Marsman (NL), Z. Pintz (HU), S. van Rijnswou (NL), and R. van Woudenberg (NL)


Each year in autumn, the EPO and the epi arrange a meeting of EQE tutors and members of the EQE Committees, usually referred to as “the Tutor Meeting”. The goals are to discuss last year’s papers, to improve future EQE’s by openly exchanging ideas and to help tutors prepare candidates for next year’s exam.

The Examination Board has kindly given the tutors permission to publish their own report of the important points so that candidates can more easily find this information. In addition, the comments can greatly assist when reading and interpreting the official Examiners’ Reports. The Tutors’ Report appears each year in the last edition of epi Information.

This year’s meeting was again held by videoconference, in the mornings of 8 and 9 November 2022. On the first day, Papers A, B, C and the Pre-Exam were discussed; on the second day, Paper D and General matters.

On the first day, only about 50 people participated of which about 35-40 tutors; on the second day, even fewer people participated. The low attendance may be due to the online videoconference format, which is less attractive and less interactive than an onsite meeting, also due to the lack of social, informal moments. The very late announcement of the dates (the meeting was only announced by 11 October, less than one month before the event and in a busy part of the year), by a message on the epi website (whereas before 2020 all tutors were informed by email by the EPO Academy or the EQE secretariat), may have played a role too. We all look forward to having the 2023 Tutor Meeting again in person at the EPO in Munich or The Hague!

Some questions for the Committees were submitted by email prior to the meeting, unfortunately also less than other years. During the meeting, additional questions were asked. The questions were addressed by the Committees and the Examination Board when discussing the papers and in the General part. The answers are incorporated in this report and can be used to supplement the information from the Examiners’ Reports.

This Tutors’ Report contains the following sections:

  1. Pass rates EQE 2022;
  2. The Online EQE 2022;
  3. General remarks from the Tutor Meeting;
  4. Paper A;
  5. Paper B;
  6. Paper C;
  7. Paper D;
  8. Pre-Exam; and
  9. Concluding remarks.

On behalf of the tutors present in the meeting, I would like to thank all the members of the Examination Board and Committees as well as the EQE Secretariat for their openness, for listening to our opinions and comments, and for providing their feedback thereto. This meeting is our yearly opportunity to learn from each other. My thanks also go to the tutors who asked questions and contributed to the discussions.

My special thanks to my co-authors -in alphabetical orderNico Cordes, Luis Ferreira, Anna Valborg Guðmundsdóttir, Andrew Hards, Harrie Marsman, Zsofia Pintz and Sander van Rijnswou for finding time to prepare the individual paper summaries.

We all wish you good luck in 2023,
Roel van Woudenberg (editor)

1) Pass rates EQE 2022

The official results for each paper of EQE 2022, as published on the EQE website and dated 7 July 2022 (one day after the candidates received their Results letters in MyEQE) for EQE2022, are shown in the table below:

Online EQE 2022* # Candidates PASS** COMP.FAIL FAIL***
Pre-Exam (4 x 70 min) 680 96,76%**** 3,24%
A (4 hours) 886 59,82% 13,09% 27,09%
B (3½ hours) 1158 77,55% 6,74% 15,72%
C (2 x 3 hours) 1400 49,14% 12,07% 38,79%
D (90 min + 90 min + 3 hours) 704***** 62,64% 8,52% 28,84%

* The Pre-Exam, A, B, C, and D papers are designed as papers of 4h, 3½h, 3h, 5h and 5h respectively [Rule 22-27 IPREE]. Since 2017, all candidates are granted an additional thirty minutes per paper to these durations [Decision of the Supervisory Board of 17 November 2016]. In the Online EQEs of 2021 and 2022, the Pre-Exam and C and D papers were split into multiple parts. Each part had to be completed before the start of the next break, with the next part only becoming available after the break. As a result, candidates were no longer free to allocate their time as they see fit across the different parts of the Pre-Exam and the C and D papers. To compensate for this restriction, the total duration of the Pre-Exam, C and D papers was extended to the lengths shown in this column.

** note: These pass rates as published do not include the results of any appeals. It is not known whether/ how many appeals have been successful in interlocutory revision by the Examination Board or before the Disciplinary Board of Appeal or are still pending before the latter.

*** The FAIL rate includes no-shows.

**** Due to the neutralization of complete Part 3 (Q.11-15) and Q.20, i.e., 70 out of 100 marks, of Pre-Exam 2022.

***** In view of the high pass rate of paper D in 2021 due to the neutralization of D1.1 because of the initial absence of the paper in English and French when the exam was started, there were relatively few resitters this year, so that the number of candidates sitting the D exam was lower than usual (usually 1000-1200, depending on pass rate of paper D the previous year and Pre-Exam pass-rate) and such that the pass-rate was expected to be higher than usual (as the pass-rate for resitters is lower than that for first-time sitters). The number of resitters was indicated to be even less than the number of fails in 2021, such that maybe some that failed in 2021 gave up for the time being or permanently. Further, all candidates were awarded full (5) marks for Q.5 of D1 2022.

In 2022, 680 candidates sat the Pre-Exam, similar as in 2021 (626), but 30% less than in 2019 (920) and 2018 (935). This low number is probably still an effect of the decisionDecision of the Supervisory Board, 20 April 2020 & Communication of the Supervisory Board, 20 April 2020 of the Supervisory Board to allow all candidates to sit main exam papers in 2021 without the need to have sat and passed a Pre-Exam, in view of the cancellation of EQE 2020 due to the COVID-19 pandemic. Of these 680 candidates, 658 (97%) passed the Pre-Exam. This pass-rate is much larger than in recent years (2021: 87% from 626 candidates; 2019: 88% from 920 candidates; 2015-2018: 74-76%), due to the neutralization of 6 questions (complete part 3 of 5 questions as well as question 20; worth 30 out of 100 marks).

885 out of 1918 candidates that took at least one paper passed the EQE (compared to 1093 out of 2780 last year). When comparing the results for the individual main exam papers with earlier years:

  • Paper A showed a significantly lower pass rate than in 2021 (74%) and 2019 (79%);
  • The pass rate for paper B was significantly higher than in 2021 (55%) and 2019 (53%) and the highest of the last 5 years; paper B shows a wide variation from year-to-year (e.g., in 2017, the pass-rate for B was 67%, in 2018 73%, in 2019 53%; 2021 55%);
  • The pass rate for paper C was similar as in 2021 (47%) and 2019 (50%);
  • The pass rate of paper D was a lot higher than usual (e.g., D 2019: 49%; 2018: 33%; 2017: 39%; 2016: 42%); this high pass-rate is largely due to a full awarding of 5 marks to question 5 in view of an error in the English version of the question (see below), but presumably also due to the relatively few resitters and due to the 20% extra time that is given in view of the split into 3 parts.

2) The Online EQE 2022

In their “Information on the schedule for the EQE 2022 examination papers” communication of 25 June 2021, updated on 2 December 2021Information on the schedule for the EQE 2022 examination papers, 2 December 2021, it was indicated how the exam papers and the schedule would be adapted for the online EQE 2022. The document provided that:

“The EQE 2022 will take place online using the same setup as the EQE of 2021 [i.e., using WISEflow/LockDown Browser]”.

“The pre-examination and both papers C and D will be split into parts. This means that candidates will not be free to allocate their time as they see fit across the different parts of the papers. To compensate for this restriction, the total duration of the relevant papers has been extended. Once the time allowed for a part has elapsed, it will not be possible to go back to that part.”

“The pre-examination will have the same syllabus and character as before, but it will be split into four parts. Each part must be completed before the start of the next break, with the next only becoming available after the break. The pre-examination lasts four hours and forty minutes. Candidates will be allowed to print the prior-art documents for the claim analysis parts before the start of the appropriate part. The documents allowed for printing will be made available during the break preceding the relevant claim analysis part.”

Paper D will have the same syllabus and character as before, but it will be split into three parts. Each part must be completed before the start of the next break, with the next part only becoming available after the break. Paper D lasts six hours.” The June version also indicated that “Candidates will be allowed to print only the calendar”, but this was replaced in the December version by “No calendars will be provided” in view of the Notice from the Examination Board of 19 November 2021Notice from the Examination Board for the European qualifying examination (EQE), 19 November 2021 (available on the EQE website under the link named “Calculation of time limits in paper D and pre-examination”) which provided that “[a]s of the 2022 examination, the practice concerning calendars will change: calendars will no longer be provided to candidates as part of the examination papers for the pre-examination and Paper D.”

Paper A will have the same syllabus and character as before [note from the editor: Paper A was not split in parts]. Paper A lasts four hours. Candidates will be allowed to print the prior-art documents and the drawings of the application, but not the letter of the applicant.”

Paper B will have the same syllabus and character as before [note from the editor: Paper B was not split in parts]. Paper B lasts 3.5 hours. Candidates will be allowed to print the prior-art documents and the drawing(s), but none of the following: the description and claims of the application, the EPO communication, the client’s letter and the amended claims.”

Paper C will have the same syllabus and character as before, but it will be split into two parts. The first part is to be completed before the break, with the second part only becoming available after the break. It will not be possible to go back to the first part after the break. Paper C lasts six hours.”
Candidates will be allowed to print everything except the claims of the patent in suit/opposed.“
The communication also included the start and end times of each (part of the) paper, and information about possible unscheduled breaks. The latter were only allowed for paper A, B, C part 1, C part 2, and D2, but not for the shorter parts (the four Pre-Exam parts and the two D1 parts).
The communication also indicated that “The documents allowed for printing will be made available approximately ten minutes before the start of the examination”. Printing was only available before entering the respective exam flow, but not anymore after entering the flow in the secure environment.

Compared to the 2021 exam, the most significant changes in the schedule were different lengths of the D1.1, D1.2 and D2 parts for D, as well as the absence of calendars in the Pre-Exam and paper D exam papers.

About two weeks before the start of the exam, in the OJ of 28 February 2022, a new version of the “Instructions to candidates concerning the conduct of the European qualifying examination” [OJ 2022, A20] was published.

WISEflow

EQE 2022 was again conducted online using the locked browser in the examination platform WISEflow. WISEflow provides a secure online exam platform, allowing candidates to take the exam from any suitable location (without any other person in the room and without any other electronic equipment in the room/within reach apart from the computer and screen used for the exam), i.e., typically from home or the office. There were no examination centers. A computer with a network connection was required, and only a single screen could be used of a size and resolution at the candidate’s choice.

For the Pre-Exam 2022, WISEflow presents each question on the left half of the screen in a language selected from the three EPO language. On the right half of the screen, 4 statements are presented in all three languages with a True/False answer option to each trilingual statement (clickable bullets). In the claims analysis parts, the prior art documents are provided as pdf documents via a hyperlink; these document could be printed before entering the exam flow.

For the main exam papers, WISEflow provides a secure environment (FLOWlock) with the paper in pdf format and a proprietary editor with basic formatting functions (headers and ToC navigation pane; underline, bold, italics and strikethrough (the latter being added compared to 2021; enumerated lists, bullet lists). The editor allowed a basic copy/paste from any text part of the examination paper into the editor and within the editor. A basic form of annotation/ highlighting was introduced in the main exam papers (an improvement compared to 2021) and it was possible in the answer in the editor and in the Pre-Exam onscreen questions. Care had to be taken with the annotation of the pdf of the paper: Only the highlights/annotations from one of the tabs were saved so that they were visible in other tabs when the paper was subsequently opened in another tab and so that they reappeared when that first tab would be closed and again opened; however, highlights/annotations done on the same exam paper opened in further tabs (e.g., on a page showing a prior art document) got lost when the tab was closed; also when a further tab was refreshed, the annotations made on that tab disappeared and a copy of the annotated paper of the first tab appeared.

As in the previous exams, candidates could bring any paper documentation, and make notes on paper. These notes could however not be handed in. During the exam, the legal texts on the EPO website were also available in WISEflow via a hyperlink “Legal texts”, including the GL/PCT-EPO and the Euro-PCT Guide; PCT legal texts were however not available as they are not on the EPO server. During the Tutor Meeting, we were informed that EPO legal texts will again be available in WISEflow during EQE 2023; availability of the PCT Applicant’s Guide is in preparationThe IB has made available a special edition of the PCT Applicant’s Guide https://pctlegal.wipo.int/eGuide/eqe/documents.xhtml containing individual annexes and the introductions to the International Phase and the National Phase as a new searchable web-application, as of 31 October 2022. Please monitor the EQE website and check the status in WISEflow. but not yet certain; other PCT legal texts seem unlikely. Candidates are recommended to check the actual situation in WISEflow during the Mocks.

Mock exams

In the first week of September 2021, it was announced“Information on planned mock examinations” dated 1 September 2021; updated on 16 November 2021 (indicating that “[T]he updated version of WISEflow will be ready for new mock exams in December”, rather than “in January at the latest” in the September version). that a first batch of mock exams would be made freely available to candidates and registered tutors on WISEflow for an extended period from mid-October 2021. These included all main exam 2021 papers, A 2019, B 2019, C 2014, D 2016, Pre-Exam 2019 and Pre-Exam 2021 as well as mock main exam papers made by epi (the same mock papers as for e-EQE 2021).

Early/mid November 2021, main exam and Pre-Exam candidates were informed by the EQE secretariat that the full and updated system would available, including invigilation, on dedicated dates and times in December, whereby candidates could test WISEflow with a set of mock examinations under exam conditions, i.e., timed examinations according to the schedule of the EQE 2022 (2 – 10 December) with video and audio invigilation (AI and human). A new set of main exam papers was provided by epi for these mocks; the Pre-Exam Mock again used the 2019 Pre-Exam. Accidentally, the C paper offered on 10 Dec was not the newly prepared one; for that reason, a new C mock was made available on 21 December. The EQE secretariat strongly recommended making use of these mock exams to test equipment and settings.

On 14 December 2021, the EQE secretariat informed the main exam candidates by email with information on the further availability of the epi Mocks, for a longer period but without any video/audio/human invigilation, as well as a date for a further mock under exam conditions (3 February 2022) to check your equipment and setting.

The latter Mocks were available until 6 March 2022, i.e., shortly before the start of EQE 2022.

The actual Online EQE 2022

The online exam took place from 8 – 17 March 2022 (Main Exam) and 18 March 2022 (Pre-Exam). Compared to earlier years, the main exam papers were in the same sequences as before (D, A, B, C), but there was always at least one day in between two successive main exam papers.

In the Pre-Exam, the sequence of the questions was randomized in each of the four parts (legal as well as claims analysis), to reduce the chance of cheating. Furthermore, in the legal part, the sequence of the statements within a question was randomized.

Legal basis in the EQE 2022

It was indicated at the Tutor Meeting that apart from Guidelines of March 2021 (as being the ones in force on 31 October 2021, according to Rules 2 and 22(1) IPREE), also the Guidelines of March 2022 (as being the ones in force on the date of the main exam papers and as such available via the hyperlink “Legal texts” in WISEflow during the exam) were accepted as a valid legal basis.

3) General remarks from the Tutor Meeting

Opening words

The meeting itself opened with words of welcome and introduction by Nicolas Favre (Chair of the Examination Board). Nicolas said that the Committees and Examination Board very much appreciate the feedback from the tutors and invited all to actively participate. He urged all participants to have the cameras on, as a discussion meeting like this does not work with people hiding behind black screens.

Nicolas expressed his thanks to everyone who supported this year’s EQE. The EQE takes an extreme lot of resource, most from volunteers. Drafting of paper is one of the most work-intensive things. Tutors responded that the effort put in the papers is much appreciated.

WISEflow

The EQE is currently in a difficult time in transition from the current, “old” EQE to the New EQE. The current system is not optimal, as we have paper papers done in electronic form. The Committees and the Examination Board are working to do it the best they can within the current boundary conditions.

Simone Fausti (EQE department) commented on the exam platform. He indicated that all feedback had been taken into consideration, but it is not always easy to accommodate for certain wishes. For EQE 2023, the format of the platform will be substantially the same as 2022. As an illustration of work that is ongoing on further improving the system, Simone mentioned that the EQE is working on the possibility to have legal basis available consistent with cutoff date, rather than -as so far- on the date of the exam. For the PCT Applicant`s Guide, an EQE compilation was frozen on 31 October 2022 and testing is progressing to make that available; it is however not yet certain that it will be available. Also for PCT Articles and Rules, the gap has been seen and the EQE is looking whether they can be made available; however, technical hurdles may hinder it for EQE 2023.

In response to a question of a tutor as to when WISEflow would become available for preparation for EQE 2023, Christoph Machwirth (EQE) answered that it became available for candidates last week and that tutors will get access soon, at least those that registered for today and that requested accessThose tutors got access on 10 November 2022, so shortly after the Tutor’s meeting.. Others can register at a later moment.

Complaints and appeals

As to complaints and appeals, Nicolas Fabre indicated that the Examination Board looks at each and every complaintRule 19(3) IPREE; OJ 2022, A20, item I.8 and appealArticle 24(3), 1st sentence REE (in OJ 2019, SE3). Each appeal is intensively discussed on case-by-case by the Examination Board before deciding on interlocutory revision or not. Complaints are discussed in groups. Complaints are often difficult to handle because they are often not complete – presumably because they need to be submitted soon after the exam; better documentation of the situation is often required.

If the Examination Board does not grant interlocutory revision, the appeal is sent to the Disciplinary Board of Appeal (DBA), without comment as to its meritThis follows the principles of Art. 109(2) EPC. All decisions from the Disciplinary Board of Appeal are rediscussed in the Examination Board. If remarking needs to be done, then the decision is also discussed in the harmonization meeting.

It was noted that appeal decisions sometimes come only a few days after the next EQE, i.e., too late to prevent an unnecessary resit. Nicolas indicated that there is a clear division of power between the Examination Board and the DBA. Whereas the Examination Board has a strict time limit for interlocutory revisionTwo months from notification of the decision – Art.24(2), 2nd sentence REE, there is no formal time limit for the DBA to decide.

It was also noted that many non-successful appeals are withdrawn at some time during proceedings and then not published. As a result, almost all published appeals are successful, which gives a misleading picture: success rate cannot be judged from the published appeals.

Outlook to 2023 [comments made spread out over the meeting by various people]

Candidates already received WISEflow access in the week prior to the tutor meeting. When registering for this meeting, tutors could also register to get WISEflow access. Those that have registered will get access within the next 1-2 weeks. Tutors that did not yet registered for WISEflow access can still do so using the registration page on the epi websiteVia https://patentepi.org/en/epi/news/a5dc5467-f8d2-4e55-a0bb-8de8df2fcecf. Candidates are strongly advised to check WISEflow and their setup using the Mocks as well as the Mock under exam conditions (including invigilation with camera and microphone). WISEflow system requirements are provided on/via the EQE FAQ “What are the system requirements?”https://www.epo.org/learning/eqe/faq.html and the recommendations via the linkAt the time of writing this article, the link is to “Lockdown computer recommendations” on https://epo-wiseflow.ladesk.com/772936-Lockdown-computer-recommendations on that page.

The UPC roadmap indicated that the sunrise period is expected to start 1 January 2023 and the UPC is expected to go life per 1 April 2023At the moment of finalizing this reports, the start of the Sunrise Period has been postponed for two months to 1 March 2023, followed by the entry into force of the UPCA on 1 June 2023. Please refer to the UPC website for the actual status (https://www.unified-patent-court.org/en). That means that Germany is expected to deposit its instrument of ratification in December, i.e. after syllabus cutoff date, so that it will not be EQE 2023.

The change of Rule 126(2)/127(2) EPC per 1 November 2023 is of no relevance for EQE 2023, nor for EQE 2024 (assuming that the REE/IPREE remain unamended until then), as the amendments is after the legal cut-off dates of 31 October 2022 and 31 October 2023 respectively. It has not yet been decided what version of the legal texts will be available online in those exams, but for EQE 2024 it would be confusing of the online version of the Guidelines is the actual version in force during the exam, which would have been amended to reflect the amended Rule, without the 10-day legal fiction for deemed delivery. It was indicated that no decision has been made yet as to how to this will be dealt with.

New EQE

The EQE 2023 and 2024 will still be according to the current format. Dates for EQE 2023 and EQE 2024 have been announced on the EQE websiteSee https://www.epo.org/learning/eqe/notices.html -> Dates for the EQE 2023 - notice of 16 December 2021 & Dates for the EQE 2024 - notice of 20 September 2022.

Tiem Reijns (New EQE WG) indicated that the “New EQE” is still being defined, with the results from the consultation being reviewed now being reviewed and implemented in the concept paper. Improvement ideas have been presented to the epi Council meeting this October. The changes that the Working Group is now working on will make the structure more easy than in the proposal used for the consultation. Only this implementation is completed, the new REE and related documentation will be prepared including transitional provisions. He indicated that it is not a workable solution to run the current and the New EQE in parallel for a long time, so that the transitional provisions will need to include exemptions. Changing the REE and the other documentation is very complicated. Once we know how future exam will be legally embedded in future REE, the transitional provisions will be drafted. The transitional provisions will be fair, candidates will not be disadvantaged. It was noted that it is not the New EQE WG, nor the EQE Secretariat, epi, the EPO, the Examination Board or the Supervisory Board that will need to adopt the new REE, but the Administrative CouncilThe current REE are in OJ Suppl 3/2019, page 1 and 217. These current REE were adopted by “Decision of the Administrative Council of 10 December 2008 amending the Regulation on the European qualifying examination for professional representatives before the European Patent Office (CA/D 26/08)” and entered into force on 1 January 2009.
The current IPREE are also in OJ Suppl 3/2019, page 18-35. These current IPREE were adopted by the Supervisory Board on 13 December 2018 and entered into force on 1 January 2019.
.

Why so few questions from tutors this year?

Tiem Reijns (Examination Board) asked why there were so few questions submitted this year and only from a single course institute. Tutors responded that it was not well known that meeting took place as there had not been a mailing with an announcement, but only a short announcements and registration page on the epi website. Also, the registration form did not include an invitation to submit questions by email or upload. It was also indicated that the online format also does not help, as it gives only very limited interaction and lacks social character. Several participants indicated that the networking aspect is completely absent in this online form and that this is a real pity. However, another tutor indicated that the online format makes participation easier as no travelling is needed, saving time and cost; this tutor suggested to have the meeting in hybrid form so as to have the combined benefits. Open discussion between tutors to pre-discuss questions is also missing in this online form. It was noted that when the Academy organized the meeting, before 2020, the announcements were done well in time and explicitly included an invitation to submit questions well in advance; this year, the Academy did not organize the meeting and the EQE Secretariat was involved only very late, while also the roles of the EQE Secretariat and epi Learning were not so clear to the participants. It was indicated that the organization will need to be improved next year and that a single organization shall be the point of contact.

4) Paper A by Andrew Hards and Anna Valborg Guðmundsdóttir

In 2022, Paper A was again held electronically within the traditional 4 hrs without being split into parts. Just before the examination, candidates were allowed to print the prior-art documents and the drawings of the application, but not the letter of the applicant.

Paper A was about a method to prepare durable paper sheets from paper pulp, which can be obtained by stamper beating of a mixture of lignin-free raw plant material and glue. A particular challenge was not only to claim the various types of inter-related independent claims but also how to actually claim them. The distinguishing features in each type of independent claim were different. The five independent claims expected this year (see Figure), could be claimed in a single application as they all related to the effect obtained by the degree of homogeneous intermixing of the glue into the paper pulp.



D1 described the traditional method of preparing paper pulp by stamper beating of cloth rags, where stamper machines have a fixed angle of 90° between hammer head and hammer shaft. D2 outlined a process for paper manufacturing from plant materials.

The technical contribution over D1 and D2 lies in making paper pulp from plant material while adding the glue during stamper beating. To ensure pulp circulation during beating, the set-up of the traditional stamper machine as described in D1 needs to be adapted to a fixed head-to-shaft angle deviating from 90°, this being the distinguishing feature of the stamper apparatus.

A discussion point during the tutor’s meeting was the “disclaimer-type” formulation of the head-to-shaft angle being “not 90°”. The distinction between this feature and arbitrarily close angles has been argued controversially in the case law. However, the EQE committee considered this to be acceptable and to provide the broadest scope, as compared to defining the angle positively e.g. between 82 and 60°, which would easily allow competitor circumvention. Fortunately, the state-of-the-art D1 has the same level of accuracy mentioning specifically the number 90°, so excluding this value affords novelty without numerical overlap.

A further critical point was that candidates should identify that the primary method claim could be drafted independently of the product stamper but then required the distinguishing step of homogeneously mixing the glue and the pulp. This could be formulated with either a displacement parameter or a viscosity difference parameter. Realising the significance of this parameter feature and defining the method without backreference to the stamper apparatus could bring an extra 10 points.

Finally, the paper sheet claim should not be drafted merely as a product-by-process claim, as the material could be defined structurally using the tensile strength parameter and the ISO norm (the norm should not be omitted). If this was not recognised 6 pts were lost for the final paper sheet product claim. The tutor’s mentioned that norms can change and should always include a date to avoid being challenged in opposition – probably this depends on the technology involved.

In addition, since this final paper sheet claim is independent, it was important to also include the essential features of “lignin-free + plant material” and “comprising homogeneously mixed glue”, with each omission losing 6 pts.

Dependent claims and the description each gave 10 pts, so these sections became critical for most candidates who missed out on all the intricacies of intermediate products, product-by-process claims and parameter definitions of the final product paper sheets. The description should focus on defining the differences over the state of the art, in particular the stamper in D1 and how the solution solves the technical problems related thereto, as well as establishing definitions of unclear terms used by the applicant.

5) Paper B by Harrie Marsman & Luis Ferrera

For Paper B, Nicolas Favre (Chairman of EQE Examination Committee I) and Wim van der Poel (Coordination Examination Committee I and member of the Examination Board) informed the attendees that the main drafter for Paper B, Andreas Böhm-Pélissier, is now a Board of Appeal member and for that reason could not participate in this meeting.

Co-drafter Ali Hijazi gave a presentation on the paper that had a pass rate of 77.55%, complemented by a compensable fail rate of 6.74%. Just like for the presentation on Paper A, the presentation on Paper B closely followed the Examiners’ Report.

Ali Hijazi noted that, although being rather mechanical in nature, this Paper B was “quite classical” and had good passing rates. There was no need to panic with all mechanical terms used, because of the explanations given in the paper and the clear guidance given by the client.

Paper B was on a snowshoe that allows wearers to downsize the deck while walking, ensuring that walking in snowshoes is as natural as possible, or for storage. In addition, the snowshoe allows its wearer to adapt the deck width and gripping properties to the snow conditions.

Ali gave an overview of the paper, discussed the prior art and official communication, and the client’s instructions and claims. Expected claims and expected arguments were then discussed. In respect of D1, it was highlighted how D1 did not disclose the specific pulley mountings (axles, sliding slots). For D2, it was emphasized that it contained explicit teaching that the deck shown therein was not combinable with D1. As indicated, this has also been covered in the very complete Examiners’ Report.

As was the case with the previous single Papers B, the points for the claims were closely coupled to the amendments of the set of claims as suggested by the client.

One of the main issues in the paper was to create novelty for the snowshoe claim over D1. The client’s proposal to combine original claims 1, 4 and 5 does not achieve this, as is also in line with the Art. 94(3) communication.

Original claim 6 is a good candidate to bring novelty. In the Art. 94(3) communication no novelty (nor an inventive step) objection is raised against this claim. Further, the client significantly indicated that he would be ready to accept some additional features of original claim 6 to be incorporated in amended claim 1. However, the client at the same time stated that the embodiment in claim 6 without the expansion control means is important for the low-cost market.

This should have brought the candidates to only add some of the features of original claim 6 in amended claim 1. Introducing also the expansion control means in an amended claim 1 resulted in a reduction of 10 (of the 30 available) marks for the claims. This severe punishment was motivated by indicating that this limitation clearly was against the wish of the client. As in previous years, the client is the authority in terms of the commercial products to be protected.

In the argumentation, it was expected to use the basis for intermediate generalization as laid down in the Guidelines H-V, 3.2.1 and applied to the contents of paragraphs [012] and [013] of the application as filed. Although the technical field was evidently mechanical, it was considered that given the robust indications, this argumentation was not problematic to candidates.

In its suggested new claim 1, the client also added the feature that the pulleys can rotate freely. However, paragraph [004] only mentioned that the wheel rotates about an axle. This limitation violates Art. 123(2) EPC and should therefore have been removed from claim 1 by the candidates. As expected, all modifications to the originally filed claims had to be discussed and basis had to be provided.

Another aspect, although “only” associated with 2 marks, was the use of the two-part form. Ali noted that within the Committee subgroup it was deemed appropriate to use the two-part form, not only because this is in the Rules of the EPC and in the Guidelines, but also because the client used the two-part form, which was deemed to express the wish of the client. In a short discussion on this point, Ali noted that for this Paper B, it was not that complicated to draft a claim in the two-part form starting from D1 as the closest prior art. In addition, it was observed that applying the two-part form in Paper B was in general easier than in Paper A.

Finally, Paper B contained some clarity issues that often occur in Paper B: process language was present in a product claim, inconsistent claim wording was used for one of the claimed features, and the claim dependency in the suggested set of claims needed be adapted (correcting dependencies usually provides significant marks while being a relatively straightforward amendment). As expected, there was no need to correct potential/hypothetical issues not raised in the official communication.

For the argumentation of inventive step of the independent claim, there was a total of 38 marks to be gained if the candidate started from D1 as closest prior art. However, starting from D2 or D3 was also possible, be it that only 25 marks were achievable in the discussion of inventive step. In addition, it was less easy to bring claim 1 in the two-part form in respect of D2 or D3.

For this year’s Paper B, it was noted that an argument in the inventive step discussion that the claimed feature “cannot be found” in the available documents was insufficient to collect the maximum number of marks.

There was a question on the use of the phrase “sliding in the guiding slots” as used in the expected claim 1. Why was that not seen as processing language? Ali commented that perhaps the use of “configured to slide” would have been better, and further pointed out that no objection to the said phrase was made in the Art. 94(3) communication. The same applied by-the-way, he said, in respect of a missing reference sign in claim 1.

As indicated above, the Examiners’ Report is quite detailed and should be read attentively.

6) Paper C by Sander van Rijswou

Hanno Schombacher gave the main presentation for Paper C 2022; introducing the topic, its problems and the expected solution. Additional remarks were made by Paolo Provvisionato. The notes below focuses on the additional information provided during the meeting, that is not in the Examiners’ Report for Paper C 2022.

Effective dates

There was no problem about priority at all. In this respect, it was an easy paper. There were not many points for the client-letter part of the exam.

Prior Art

Annex 2. This prior art was an internet newsletter. Relevant is to discuss how it is disclosed? You had to look at the guidelines. It was considered publicly available from the day it was assumed posted. Relevant pointers to this included the header of the newsletter. You could derive the publication date.

Annex 3. A3 was more complicated for the date, it is not so clear when it was disclosed. In the document balls are talked about that were used in the past. Important in this document, is that two different prior uses were substantiated. The two embodiments should be distinguished by a candidate.

This type of disclosure, with two different pieces of prior art in a single reference will be in future C papers again.

Annex 4 was a USB stick with slides that were shown at a presentation

Annex 6 is a 54(3) document. How can you use that specific information? You can only use it for novelty, or general technical knowledge

Claim 1

Lack of novelty with respect to A5 was expected. A5 clearly shows all the features apart from the product by process feature ‘whereby a void is formed between the inner strand and the outer layer by removal of material using a solvent’. You need to identify that this is a product by process feature, and that there is a guideline section for that.

A5 does not use a solvent but heat. It had to be argued, that in A5 the removal creates a void. This was not seen by all candidates. The solvent also creates a void, this is in A4. Since both processes have the same result—the partial removal of material—there is no structural difference.

There was no information that a void created by a solvent is different from a void created by thermal treatment. Any idea that there could be a difference, comes from hindsight by the candidates. This is invalid information. The information in the paper, was simply that there was partial removal of material, and that there is a void.

Later the novelty of Claim 1 was further discussed after a question. Although the novelty of claim 1 could be defended by the proprietor, the main skill is making the best out of the material that we have. That is a basic skill for a patent attorney. In real life, we find ourselves attacking a patent, with even less good material than is presented in paper C, and we have to do our best to attack the patent. The way the paper is constructed, obviously it is not perfect. There will be things that do not perfectly match the solution. Otherwise, the paper would become a crossword puzzle. Since, at least, it could be debated whether the product by process claim is novel or not, it is a good opposition position.

The alternative attack on claim 1 is an inventive step attack. This attack could give you nearly full marks if a candidate correctly identified all the features. A few marks are lost for not identifying the product by process features.

It was also expected to point out the difference between the phrases (suitable) ‘for’ and ‘such as’ in Claim 1. There were marks for that.

Claim 2. Features of claim 1 are included in the method of claim 2. This makes it clear that A5 has to be the closest prior art.

Claim 3. Added subject matter. The claim as filed was split into two parts. Was there any basis for this isolation of features? If you isolate the features, information is missing. The claim becomes more generic. Other embodiments are included that are not disclosed. No reference to the guidelines was needed.

Claim 4. Two attacks were expected. The claim lacked novelty with respect to A6. The second attack was lack of inventive step. Some candidates missed the novelty attack. The bladder of claim 4 was implicitly disclosed.

Claim 5. The difficulty was the closest prior art. Both prior uses of A3 comprise a goal, and ball. So they were not so different in this respect. However, the second ball in A3 is a further development of the first ball. The skilled person would not change a further development back to the previous model. There is no incentive to combine the second, further developed ball, with the first older ball.

Starting from the second prior use was considered less convincing. Why would a skilled person modify the second prior use with the first prior use? Moreover, they both have the problem of unreliable goal detection.

Many candidates mixed the two prior art embodiments, sometimes not even distinguishing the two embodiments at all.

Claim 6 was not liked by the candidates. The claim was a computer implemented invention which caused problems for some candidates

It was the first time that a mixed-type invention was used in the exam. Candidates were expected to use the COMVIK approach, well-documented in the GuidelinesGuidelines G-VII, 5.4 on “Claims comprising technical and non-technical features”. Note that in the 2022 edition, Example 4 in G-VII, 5.4.2.4 was replaced in view of G 1/19 and a new Example 5 was added in G-VII, 5.4.2.5 which explicitly and in detail applies the steps of the problem-solution approach according to COMVIK..

The claim itself is technical, it is only that there is a non-technical feature. Candidates should identify the features that contribute to technicality.

Often candidates simply said that it was completely non-technical. They did not identify the technical features. You have to apply the guidelines, which are quite comprehensive. There were several hints that the feature in Claim 6 was not technical: the reference to business in A1, and to commercial in A3.

A normal set of claims in any technical field now often comprise a computer product claim. This is quite usual. So if the paper is to test whether a candidate is fit for practice, the paper has to include such a claim. An attorney should know how these claims are considered by the EPO.

It should be assumed by candidates that this kind of claim will occur more often in the paper.

Incorrect attacks

A question was asked about incorrect attacks. At a previous tutor meeting it was suggested that these were included in the marking scheme to ensure uniform marking.

Paolo Provvisionato explained that incorrect attacks are not included in the marking scheme, but that alternative attacks are. In the pre-marking meeting, a good sample of papers are looked at to see if there are trends in the answers that require attention. Sometimes candidates spot differences, or mistakes, or misunderstandings. Alternative answers that are not considered model answers, but those that deserve consideration, are considered because they fit in the definition of fit for practice.

For example, this year it was noted that the product by process features were difficult for some, as they can be debated. Some people did not follow the approach in the Examiner’s report. However, they lost maybe a couple of points, assuming they attacked the claim with some sense. Since Claim 1 could have been attacked differently, the candidates that did so in the proper way were surely successful.

7) Paper D by Roel van Woudenberg and Zsofia Pintz

The D committee was represented by Tiem Reijns (epi, chairman D, member Examination Board), Markus Markmann, (EPO, D1) and Gabriele Gislon (epi. D2).

General remarks (Tiem Reijns)

Paper D 2022 was generally well-scored: the pass-rate was 62.6% and 8.5% compensable fails. Candidates seemed well prepared, but there were very few resitters and first-time sitters generally score higher than resitters. It is expected that the pass-rate goes back to 40-50% (as before) when the number of resitters increases.

The electronic syllabus was used widely. Also, the answers contained a lot of copy-pasted parts from the Guidelines (sometimes entire paragraphs), which is helpful in correcting, but not really showing understanding and hence not attracting any marks if not applied to the facts of the case; a mere copy is not considered an answer.

The forced time allocation caused candidates to prepare better (and score better) on Part I. Fewer candidates than before skipped entire Part I questions. Part 2 was relatively straightforward with a straight timeline, but it was not easier than usual: candidate missed a lot of aspects in their answers.

There was a problem with D1, Q5 for which all candidates received full marks; this also had a small positive effect in the pass-rate. In the English version of D1, Q5, a document date was given as 2017 and hence clearly pre-published, while it was given as 2019 in the German and French versions, i.e., in priority period so that a careful (partial) priority assessment was necessary. In view of this error and in order to provide a level playing field and equality amongst candidates (D 11/19 of 26.04.2021, online on 14.06.2021), the Examination Board decided to neutralise D1, Q5.

The error was indicated to be a human error. The paper was checked by several people, in all three languages, no one had spotted the error. If the meetings would have been in person, it would almost certainly have been spotted.

A tutor asked whether why the Committee or the Examination Board had not informed all candidates during the Exam that there was an error in the English version and how it had to be corrected, so that the exam could have continued, as was done in paper times, such that there would not have been a need for any neutralization. Tiem indicated that this was a small question, such that changing the question on the flight could have made people that already started with answering it lose time. Also, D1.1 2021 learned that giving extra time in the WISEflow system also does not always work and has complications.

General – REE/IPREE
The paper length was similar as in the last 5 years, with 30 min extra time for revision and another 30 min extra due to reduced flexibility on allocating time. From the submitted answers it seemed there was no serious time pressure in Paper D 2022.

Only legal basis from the syllabus in the REE/IPREE is legal basis. However, alternative legal basis is often accepted too: sometimes from other parts in syllabus, often also other documents as long as totally relevant and fully covering the scope.

Candidates using the latest Guidelines rather than those on 31.10.2022 were not be penalized, as the online version that is offered is the latest one. However, it is advised to stick to the syllabus. If a newer version is used, it is recommended to indicate that the newer Guidelines were used.

Examiner’s Report, Answering and Marking
The purpose of the Examiners’ Report is to help future candidates prepare. The Examiners’ Report shall be read as the correct factual answer for 100 marks. In some questions alternative answers attracted marks, but only the best answer is in the Examiners’ Report. Some additional comments were awarded extra marks (sometimes referred to as bonus marks).
In principle, all information in a question is relevant. Candidates should answer the question based on the information provided and shall not speculate. Giving both a correct and a wrong answer to the choice of the marker, will NOT attract marks. Every word is there for a reason. Focus on trigger words when preparing your answer.
Full legal basis is what is needed to support the answer in full -Article AND/OR Rule AND/OR Guidelines AND/OR case law-, whatever is needed to support all aspects of the answer. Alternative legal basis often attracts (full) marks.
Including the marking table was overlooked in the compendium. A corrected Examiners Report on the EQE website now.
The split of marks for D2, Q1 over the sub-questions a-e is not provided in 2022, as it was felt overdone to split the (only) 22 marks for 5 sub-questions into individual marks for the sub- sub-questions.

D1-part: summary of the paper

This year’s D1 came in two parts: 1 first part of 1 h 30 min and 26 marks (D1.1) and a second part of 1 h 10 min minutes and 19 marks (D1.2), i.e., a total of 45 marks. The D1 had a well-balanced mixture of EPC and PCT questions, with common topics as well as some less familiar topics.
Candidates that were well-prepared, with a sound legal knowledge and familiar with their legal reference books and other material should have been able to score 50-60% or more out of the 45 marks within the time available.
Some topics were common topics that candidates could have expected, such as remedies (here, re-establishment in the further processing period), fees, priority – even though it was surprising that partial priority was again tested (also already in D2 2019 and D1 2021 –, dealing with lack of unity, procedural and substantive aspects of opposition. Some topics were not so familiar, e.g., missed renewal fees that fell due in between when loss occurred due to missed FP correction and the decision on reestablishment, or amendments at a very late stage of examination.

The online access to the Guidelines GL/EPO and GL/PCT-EPO in html-form (only) was convenient and may have been of additional help.

The first D1 part, D1.1, consisted of three questions of 8, 10 and 8 marks, i.e., a total of 26 marks, for which 1 h 30 minutes were available, followed by a break.
In Q.1, the time limit to respond to an office action, as well as -after analyzing the case- that for its further processing was missed. It was asked which steps need to be performed to ensure that the prosecution continues and by when. The answer had to include an analysis of the situation as-is, and re-establishment of the further processing period, and remedying two missed renewal fees using Rule 51(4) EPC.
In Q.2, the applicant asked you to correct a spelling mistake in one of the claims, the correction of which is obvious, while the period to respond to the Rule 71(3) communication is running. Also, he wants a grant as soon as possible and the granted patent to take effect in the Netherlands. Candidates were expected to file reasoned corrections under Rule 139/Rule 71(6), respond a.s.a.p. to the next Rule 71(3) (some candidates incorrectly still proposed a waiver), pay the next renewal (with surcharge), and file translations of the claims and pay a national fee to the NL office.
Q.3 related to a PCT application with two inventions lacking unity, of which only one was claimed. In a first sub-question, it asked how to request SIS and whether that could also include the second invention, requiring a candidate to address all details of PCT Rule 45bis. In a second sub-question, it asked whether the second invention could be examined in the European phase, requiring amending to the second invention when/shortly after entering the European phase and paying an additional search fee in response to the invitation under Rule 164(2) that the Examining Division would issue.

The second D1 part, D1.2, consisted of another three questions of 6, 5 and 8 marks, i.e., a total of 19 marks, for which 70 minutes were available.
Q.4 related to an EP application to a new raw material, but which did not describe a method for obtaining it. Instead, it referred to the relevant part of an earlier, non-published application. Candidates were asked to discuss whether the product was sufficiently disclosed, and had to check the requirements in Guidelines F III, 8 / H-IV, 2.2.1. Candidates also had to recognise that, as a result, the application did not give rise to a valid priority for the product, whereby a subsequent application that invalidly claimed priority from it was not new with respect to a 54(3) prior right.
Q.5 has two aspects: 1) a partial priority situation where a first conceptual part of the claimed range benefits from priority but the remaining conceptual part does not, and 2) a clearly erroneous disclosure, prior art to the remaining conceptual part only, for which it is clear to the skilled person what it should be, so that it is considered to contain the correction (Guidelines G-IV, 9 (i)), and 2). When combining the two aspects, a candidate would need to conclude that the claim is new.
In the English version of the exam paper, the disclosure was accidentally mentioned in 2017 rather than 2019, whereby the priority analysis became unnecessary and the claim trivially not novel. Due to the difference with the (correct) German and French version, the Examination Board decided to neutralize this question and award the full 5 marks to all candidates.
In Q.6, two scenarios were presented. In the first scenario, an opposition was filed based on a 54(3) prior art that disclosed claim 1, but not claim 2, of the granted patent; the sole opponent withdrew the opposition the day after the opposition period expired. In the second scenario, the opposition was rejected and the sole opponent filed an appeal based on that same document; the opposition was subsequently withdrawn. For both scenarios it was asked how the opposition would continue and what the proprietor can do, before the EPO, to address the lack of novelty. Continuation of own motion and requesting maintenance in amended form was expected for the first scenario, and immediate termination of the opposition/appeal and request for limitation under Art.105a for the second scenario.

The full answers to the questions are given in the Examiners’ Report. It also indicated, as every year, important guidance for answering (e.g., “Candidates are reminded that they should pay attention to the way the questions are asked”) - most of these points were also emphasized at the meeting (see above under “General remarks” and “Answering and marking”).

D2-part: summary of the paper

This year’s D2 was a 55 mark paper for which 3 h 20 min was available. It was somewhat more difficult than D2 2021, but somewhat less complex than the D2s of earlier years. The paper presented all inventions and applications in a clear and concise way, including acronyms for the various claims features and including effects of all features to support inventive step. The questions were very explicit and were, as in most recent years, a clear “agenda” as to what to address.
Well-prepared candidates, that did not only give their conclusions but gave a complete reasoning without any implicit steps, should have been able to address most of the expected aspects (but will also have missed some aspects or some detail) and to score 50-60% or more out of the 55 marks within the time available.

The paper had to be taken fully from the screen in WISEflow - nothing was printable. The pdf of the paper could be viewed side-by-side with the editor, without an annotation possibility; or in one or more separate tabs, with limited annotation possibility. For the first time, no calendars were given with the exam paperNotice from the Examination Board for the European qualifying examination (EQE), 19 November 2021 (available on the EQE website under the link named “Calculation of time limits in paper D and pre-examination”), but candidates had to bring their own lists of Saturdays and Sundays and had to check EPO closure dates in the OJ.

As usually, the paper required a careful analysis of a plurality of EP and PCT patent applications from the client (OTP) and a competitor (TOR). Also, there is another, (possible) friendly party (Avidus). The various applications described and/or claimed one or more of a) a FEED machine with a feeding duct, b) a FEED machine with two feeding ducts (FEED-2), c) a CLEAN method (usable with a FEED-2 machine), d) a SPRAY device, and e) a FEED machine with a SPRAY device. The first question explicitly asked to address each of these in turn.

A mix of common and less-common aspects of a D2 paper were to be considered: non-patent disclosures: a demonstration at a fair, a brochure, exchange of technical documents and a prototype in the presence of a secrecy agreement, a video on the client’s website; multiple applications from the same application claiming the same subject-matter - careful check of first application requirement/issues; differences between a subsequent application and its priority application; interpretation of “a” (vs “one” and “two”) in a claim - novelty, scope of protection; product and method claims; Art.54(3) prior art, also vs the European phase (only) of a PCT-application; non-claimed subject-matter; entitlement; the other party just received a R.71(3) communication for an application with a broad claim scope - how to reduce/remove the risk; and different territories: EP and non-EP (US, IN).
Also, some suggestions and explicit requests from the client has to be addressed, e.g., a suggestion of the client to submit recently obtained evidence of a new effect in support of inventive step in defending his patent in opposition - why/ why not follow the suggestion; in respect of a pending opposition filed by a German lawyer with the need for a response to the notice of opposition, the client suggests to submit that the lawyer has no interest in the case so that the opposition must be rejected; opponent argues lack of novelty w.r.t. a video on the client`s website and lack of inventive step; intervention in a pending opposition appeal, while the client only received a “cease and desist” letter from the proprietor and while the opponent/appellant is considering to withdraw the appeal (if they can make a deal with the proprietor rather than with the client) - based on a new ground of opposition; and how to be attractive for an investment fund that can help the client to manufacture their own machines and to expand in US, CN and IN.

This was one of the rare D2-papers where you do not need any licensing or cross-licensing! It may have cost candidates significant time during the exam to confirm and feel confident that this was indeed the case.

Comments from Committee

The Committee commented on the questions submitted prior to the meeting and addressing various specific items in D1 and D2 questions.

Questions in the D1 part (presented by Markus Markmann)

A tutor requested a clarification of the interpretation given by the D Committee on Rule 51(4) EPC, in the case of re-establishment in respect of the further processing period. In particular, it was asked whether the relevant loss of rights in the Rule was the deemed withdrawal due to the missed period to respond to the Office Action, or the loss of the right to revive due to missing the period for further processing, where the latter period is the one remedied with re-establishment. If the first, both missed renewals would be in Rule 51(4)(a); if the second, the 5th year renewal would be in Rule 51(4)(b) and the 6th year renewal in Rule 51(4)(a). The tutor noted that the scenario of this two-stage loss is not documented in any section in the Guidelines in the context of Rule 51(4)(a) or (b), not in any case law.
Markus answered by providing, as legal basis for date of loss of right: G 4/98, r.3.3 and G 1/90 r.6: “3.3 What has been said so far indicates that under the EPC there is no retroactive effect where the EPC uses the term “deemed to be withdrawn”. See also G 1/90, where the Enlarged Board of Appeal stated in the case where the EPC deems the application to be withdrawn that “the loss of rights occurs on expiry of the time limit that has not been observed.” (point 6 of the Reasons), and J 4/86 (supra, point 1.1).” Consequently, the loss of rights occurred on expiry of the time limit set by the communication under Art. 94(3) EPC, i.e. on expiry of 11 Jan 2021. The loss of right at expiry of the further processing period is a loss of right to file a further processing request, but not resulting in the application being deemed to be withdrawn, which is required by Rule 51(4). Also, no further loss of right occurs due to non-payment of renewal fees (OJ special edition 5/2007, page 92, chapter III, comment to Rule 51(4) EPC). Hence, the sole relevant loss of rights occurred on expiry of 11 Jan 2021, i.e. before due date of 5th renewal fee, whereby Rule 51(4)(a) is applicable.

The possible solution mentions “Article 1(2) and Article 1(3) of the London Agreement” as legal basis for “To have the European patent based on EP-B take effect in the Netherlands, file a translation only of the claims in Dutch”. Markus indicated that Nat.Law Table IV (or Nat.Law Table IV, column 3) were also accepted, as alternative legal basis to Art. 1(2) & (3) London Agreement, without a loss of marks.

The model solution does generally not include possible alternative legal basis. It was indicated that alternative legal basis is legal basis that has the same content as the model solution. For example, where the Model Solution cites a section from the Guidelines, which refers to a T decision and provides a reasoning based on that decision, citing the T decision as alternative to the Guidelines is also fine as ling as the argumentation is used.

A tutor commented that at earlier occasions, the D Committee indicate that there is a trigger word in the question for every aspect that is to be discussed in the answer. The tutor asked what the trigger word was in Q.2 for discussing the renewal fee. Markus commented that the D Committee has been very consistent in indicating that every word in the question has a reason/meaning. Markus answered that, for the renewal fees, the “trigger word” is the filing date.

The D Committee confirmed that a mere copying of a Guidelines section, a mere copying of a headnote or reason from a decision, or a mere copying of a part of the Case Law Book, does not result in any marks at all, even if the relevant passage is the correct passage that should be applied when answering a certain question. To score marks, the context and relation to the question must be clear.

A tutor noted that, when comparing the number of statements /words per mark for Q.6 to that of Q.1, Q.2 and Q.3, it seems that a significantly longer answer was needed for full marks for Q.6 than for the other questions. The tutor asked whether the number of marks available for Q.6 was initially larger but was it reduced to get the total number of marks for this D1 to 45. Markus answered the number of marks is proportional to the complexity, which is not necessarily proportional to the number of words of the question. The indication how complex an answer is given by the number of marks indicated in the header of the question.

Questions and comments w.r.t. the D2 part (presented by Gabriele Gislon)

Gabriele Gislon, co-author of the D2 part, discussed the 2022 D2 part. He indicated that the paper had four main topics: 1) Analysis of a complex patent situation, 2) applicant not entitled to the application, legal remedy, 3) novelty-only prior art generated by overlapping applications of the same applicant, and 4) admissibility of new grounds in appeal proceedings, legal remedy via intervention. Hence, for solving two of the problems, some kind of national proceedings were necessary:
1) The bad party (competitor) had stolen the invention and filed an application to which he was not entitled. Clear from contract etc;
2) Necessity to have fresh grounds in appeal, which was only possible via intervention. So national proceedings as basis for intervention in appeal had to be addressed.

A tutor asked whether Art. 55 had to be addressed in Q1 or Q3: as-is vs. improvement. The tutor noted that Art. 55 had in the past always been an improvement topic (Q.3) and never an as-is topic (Q.1). Gabriele answered that the analysis of the situation as it stands includes that TOR-EP is per se non prejudicial to novelty of OPT-EP1. This is a fact rather than a discussion of improvements. Further, because of the video prior art, informing the EPO would not have been an improvement and would thus not be required in the answer to Q3. The Committee emphasized that it is important that the candidate compares the new situation with the as is situation. For this year’s solution it fitted better in the way it was published, in the Committee’s opinion. However, a candidate who wrote it elsewhere in their answer, would have received the points. In response to a tutor asking whether it matters where the answer is if as-is and improvement are asked in separate questions, Tiem indicated that it does not matter: the Committee gives the points irrespective of where the discussion is in the answer.

It was commented that this D2, as well as that of 2021, seemed to be relatively easy. Gabriele answered that the Committee spends a lot of time to trim the first draft of the paper to make it as clear as possible in the shortest possible time. He also noted that this paper had quite some complexity, requiring required knowledge of several different issues. The Committee was happy that the paper was seen as easy to understand. For the next D2, the Committee again strives for an easy to understand paper with, again, a reasonable difficulty level.

It was also observed that, if candidates had not noted the Art. 55 issue, the video prior art discussion would still allow candidates to score marks for discussing that claim. Also, the video prior art rendered the conclusion that the claim is not new, important for the rest of the paper, independent of an Art.55 discussion. A tutor commented that designing D2 papers to prevent knock-on effects of a single error or oversight has been consistent practice for many years and is much appreciated.

8) Pre-Exam by Nico Cordes, Andrew Hards and Roel van Woudenberg

Examination Committee IV (Pre-Exam) was represented by Stefan Götsch (EPO; general part) and Volker Franz (EPO; specific questions/statements)

We will first give some general comments on the paper, then discuss the legal part of the paper, followed by the claims analysis part, and then discuss the session at the Tutor meeting.

General remarks

This was the second online EQE with the Pre-Exam being held in a similar way as in 2021: the Pre-Exam was split into 4 parts and the questions had to be taken almost fully from the screen (only the description of the application and the prior art in the claims analysis parts were printable). Each part had to be completed before the start of the next break, with the next set of questions only becoming available after the break in between two successive parts. The split into parts means that candidates were not free to allocate their time as they see fit across the different parts of the paper. To compensate for this restriction, the total duration of the paper was extended: for every part, an extra 10 minutes was available so that each part lasts 70 minutes. Once the time allowed for a part had elapsed, it was not possible to go back to that part. The Pre-Exam 2023 will have a similar design.

In the legal as well as the claims analysis parts, the order of the four statements in each of the questions was randomized, i.e., it was different for different candidates. Also, the order of the questions was randomized in the legal parts (e.g., the sequence used below is the one shown in the CompendiumCompendium: Pre-Exam: https://www.epo.org/learning/eqe/compendium/preexamination.html), but not in the claims analysis parts.

All questions, statements, prior art documents and claim sets were provided in all three official EPO languages. Other than in Pre-Exam 2021, candidates could view the question body in a single language of choice in a first column on the left half of the WISEflow window. However, the statements were presented in a second column on the right half of the window in all three languages without the possibility to select just one: this resulted in quite some scrolling, especially in the claims parts, where a single webpage showed four tri-lingual statements with True/False tick boxes.

Within a part, candidates were free to take the questions in any order. After finalizing a part, it was no longer possible to go back to that part to make any changes.

The online Pre-Exams have the same syllabus and character as the earlier paper exams, and candidates are -as before- allowed to bring any printed reference material. During the exam candidates also had access to the EPO Legal Text pages, so including the full EPC Articles and Rules, Guidelines, GL/PCT-EPO, National Law Tables, Case Law, and the Euro-PCT Guide (HTML versions), but not to the PCT Legal Texts (they are not on the EPO domain, but on the WIPO domainPCT Resources on WIPO domain: https://www.wipo.int/pct/en (English), It was indicated that candidates need to monitor the Notices on the EQE website to check what is available online in Pre-Exam 2023.) and not to the full pdf-versions. Access was to the live versions, so to the versions in force on 18 March 2022 (so not the version of 31.10.2021 acc. Rules 2 and 22(1) IPREE) and answers based thereon were also acceptedFor the questions of Pre-Exam 2022, the differences between both versions of the Guidelines had no effect on the answers..

The exam had a mix of topics (common and some less common), with some surprising absences: no time limits, no remedies, no EP entry, no 71(3) acts, no priority. Entitlement was certainly unexpected for many candidates, as well as some elements of the appeal question and details on (late) fee payment under PCT.

Unfortunately, complete part 3 (Q.11-15) as well as Q.20 had to be neutralized due to errors in (some language versions of) the paper, i.e., 30 out of 100 marks were awarded to all candidates irrespective of their answers. The lack of any further neutralizations (e.g., statement 1.3) and/or specific neutralizations in part 3 (e.g., statement 11.3) may be a side-effect of this, as the pass rate was already above 96% due to these neutralizations: only 22 (including no-shows) out of 680 candidates failed.

At the Tutor Meeting, it was indicated that all information that was brought to the attention of the Committee and the Examination Board, as well as posts and comments on blogs, were carefully considered to see whether any neutralization were required. However, apart from a complete neutralization of Part 3 (Q.11-15) and Q.20 (herein, the numbering follows that as presented in the Compendium-version of the paper) due to translation/ drafting errors, no individual statements were neutralized. In view of the high pass rate, it seems unlikely that some appeals may still pending. However, no comments could be made by the Committee and Examination Board at the time of the meeting on possible pending appeals nor on possible concluded appeals.

Legal part

The legal part of this year’s Pre-Exam had similar style and a difficulty level as in 2021 and 2019, and a bit more difficult than those of 2016, 2017 and 2018. As in the last few years, the legal questions were a mix of questions addressing several topics that were to be expected (such as filing date requirements, languages, representation, late submissions in opposition) as well as several less standard topics (such as entitlement, composition of the Board of Appeal, payment of international filing fee with late-payment fee, transfer of opponent status) which well-prepared candidates would have been able to find in their EPC/ PCT/ reference materials/ Guidelines, albeit with some effort. Surprisingly, not a single question addressed time limits, neither under the EPC nor under the PCT, while it was the first exam for which no calendars were provided with the exam paper and the Examination Board had emphasized prior to the exam that candidates should bring their own list of Saturdays and Sundays and should be able to look up the closure days in the OJs of the last few yearsLink “Calculation of time limits in paper D and pre-examination” on EQE website to “Notice from the Examination Board for the European qualifying examination (EQE)” dated 19 November 2021.. The most recent G-decisions, notably partial priority (G 1/15), were not yet tested in this Pre-Exam (while G 1/15 was already tested in D papers of 2019, 2021 and again this year, 2022). Additionally, no questions dealt with priority as such this year.

Well-prepared candidates having good knowledge of the EPC and PCT and knowing their material well (reference books, EPO Guidelines, PCT Applicants’ Guide, Euro-PCT Guide, our course material such as our flowcharts) for fast look-up should have been able to answer most of the statements correctly and well within the time available, provided they had their material updated to the legal status of 31 October 2021 (the cut-off date for EQE 2022). The EPO Guidelines in particular play a key role to get to the right answers. Candidates with a poor preparation will have found those questions considerably more difficult than the questions from earlier years.

Q.1 addressed key aspects of entitlement (applications filed by non-entitled persons) and stay of proceedings, a rather unexpected topic in view of the level expected from candidates when sitting the Pre-Exam. The question addressed the requirements for requesting a stay, effects of the stay, renewal fees during stay and options after a successful national decision recognising entitlement, and could be answered using Guidelines A-IV, 2.2 on “Stay of proceedings for grant” and its subsections.

In Q.2 the topics: own motion of the Opposition Division, fresh grounds, third party observations and limited extent of opposition, were addressed, testing the candidates understanding of G 7/95, G 9/91 and G 10/91 and/or Guidelines D-V, 2.1-2.2 and E-VI, 3. The main aspects which were tested were the extent of the opposition and the competence of the Opposition Division.

Q.3 tested various aspects of appeal, and was considered difficult by various candidates. It was one of the few legal questions where the answers had to be found in the EPC itself (Art. 21(2)-(4), Art. 111 EPC) rather than in the Guidelines. The questions focused on the composition of the Board of Appeal, remittal and fresh grounds for opposition as a basis for the Board´s decision.

In Q.4, transfer of a European application was tested (for specific designated states, European Patent Register), similarly as in an earlier Pre-Exam. Candidates may not be very familiar with transfers from their daily practice and may thus consider this a difficult question. Note however, that the question can be fully answered using Guidelines E-XIV, 3 on “Transfer of the European patent application” and E-XIV, 4 on “Transfer of the European patent”, so that a candidate that is well familiar with the EPC and his/her reference material can nevertheless answer this question efficiently.

Q.5 addressed various aspects of filing of a direct European application and language of proceedings (reduction of filing fee, filing date, translation and (no) change of language of proceedings), and should not have given any difficulty to a well-prepared candidate.

Q.6 addressed various aspects of filing a divisional application (filing offices, language of filing, timing aspects), and was also quite straightforward, using, e.g., Guidelines A-IV, 1.

Q.7 was directed to first and further medical use of a known product (molecule). Not all candidates are familiar with such medical use/indication claims from their daily practice, but the special provisions of Art. 54(4) and (5) EPC in view of novelty for purpose-directed product claims for first and further medical indication are considered a key element of the EPC, so that all candidates need to understand and apply these provisions. They were also tested already in an earlier pre-exam and are well documented in Guideline G-VI, 7.1.

Q.8 was possibly the most difficult question. It addressed the payment of the international filing fee without and with the late-payment fee for a national and resident of the USA: where to pay the corresponding fees and what are the consequences of non-payment, as well as who will notify the applicant or the designated office that the application is deemed to be withdrawn (resp. rO [PCT Rule 26bis.1(c)] and IB [PCT Rule 29.1(ii)]).

Q.9 addressed more aspects on the filing of a direct European application (filing date/ language of filing, request for grant form, representation for filing) as well as on filing an international application (submission of claims on filing). Candidates may have overlooked that one statement was on an international application (claims for filing date) while the other three were on direct EP applications and would have lost 2 marks by answering all statements as if they related to direct EP applications.

In Q.10, various post-grant topics were tested: correction of errors in the B1 specification, grounds of opposition, language of opposition and transfer of opponent status. The wording of statement 10.4 was slightly different in the original Compendium than in the real examIn the exam itself, statement 10.4 read: “To transfer the status of opponent to a different person during opposition proceedings, it is sufficient to file a declaration including the names, addresses and signatures of both the original opponent and the person wishing to take over the status of opponent.” In the exam as published later in the initial version of the compendium, and in the initial Examiners’ Report, it reads: “In order to request a transfer of the status of opponent to a different person during opposition proceedings, it is sufficient to file a declaration including the names, addresses and signatures of both the original opponent and the person wishing to take over the status of opponent.”, i.e., the opening words differ between “To transfer the status of opponent” and “In order to request a transfer of the status of opponent”., but this did not lead to a neutralization – at the meeting, it was indicated that the original Compendium included the wrong text and that it was corrected in an updated version of the Compendium as soon as that was discovered.

Claims analysis part

The claims analysis part of the Pre-Exam 2022 had a similar style as in 2019 and 2021, in that the claims analysis part was structured into two parts which were in the 2022 exam separately identified as part 3 and 4 (with parts 1 and 2 being the legal parts). Parts 3 and 4 each concerned a separate case with their own set of questions, rather than one case of 10 questions as in the exams before 2019:

  • Part 3: A yoga mat and a textile bag for a yoga mat (questions 11-15)
  • Part 4: Preserving an alcoholic beverage in a bottle (questions 16-20)

Each of the parts 1-4 was given their own timeslot with each timeslot being divided by a break of 20 minutes (between parts 1 and 2, and between parts 3 and 4) and a larger break of 80 minutes between the legal parts and the claims analysis parts.

Part 3 dealt with the topic of a Yoga mat.

However, there was a translation error in the German version [005] of the description, second line, where it should read Poly-Y and not Poly-X:

English version (correct)

[05] In an embodiment, the second face contains a material which reduces the generation of sweat on the athlete’s body. We have found that poly-Y is a highly suitable material, since it provides improved reduction of sweat generation on the body of the athlete while the athlete is in contact with the mat. …

German translation (incorrect)

[005] In einer Ausführungsform enthält die zweite Seite ein Material, das die Schweißbildung im Körper des Sportlers reduziert. Wir haben festgestellt, dass Poly-X ein sehr geeignetes Material ist, denn es sorgt für eine verbesserte Reduzierung der Schweißbildung auf dem Körper des Sportlers, während der Sportler Kontakt zur Matte hat. …

Due to this translation error, the Examination Board decided that part 3 was to be fully neutralized, meaning that each candidate was awarded the full 25 points (5 questions of each 5 points) irrespective of their answers and of which language was used.

Otherwise, notable aspects of part 3 included the following:

The prior art documents D1-D4 were presented as 1 paragraph each, thereby fitting all prior art documents onto just a single A4 page. This conciseness should have helped candidates in reducing the reading effort and thereby the complexity of this part.

The claim set of 14 claims contained several independent claims, namely an independent claim on the yoga mat, an independent claim on a textile bag for covering and transporting yoga mats, and an independent use claim.

Substantive topics in part 3 included clarity, scope of protection, novelty, inventive step, and support for amendments. Of particular interest were:

  • In Q11 a statement which tested whether candidates would recognize a functional feature, and a statement which tested that an indication of a purpose in a claim on a physical entity will limit the claim to a certain degree, namely by requiring the physical entity to be at least suitable for the stated purpose.

  • In Q12 a statement which questioned whether a particular claim was novel over D1-D4, requiring candidates to perform four novelty assessments for a single statement (as the claim indeed turned out to be novel over D1-D4).

  • In Q14 a statement about the clarity of a claim with an unusual parameter.

  • In Q15 a statement which concerned the novelty of a use claim, where the claimed use was implicitly disclosed by D1.

Part 4 dealt with the topic of preserving wine.

Notable aspects of part 4 included the following:

The prior art documents D11-D13 were presented as 2 paragraphs each, so more extensively than the prior art in part 3 but nevertheless very concise compared to previous pre-exams.

The claim set of 9 claims included two independent claims, namely a method for preserving wine in a bottle and a method for more generally preserving an alcoholic beverage in a bottle. The latter independent claim included 7 dependent claims.

Substantive topics in part 4 included clarity, scope of protection, novelty, inventive step, and support for amendments. Of particular interest were:

  • In Q16 a statement in which candidates were required to recognize that a stated purpose (“for preserving wine”) is a limiting feature in a method claim and that the prior art, which does not disclose this purpose, is therefore not novelty destroying.

  • In Q17 a statement on the novelty of a claim 5 which was dependent on claim 3 or 4. However, in the dependency on claim 4, there was technical contradiction in the claim which made judging the novelty of claim 5 when dependent on claim 4 difficult. However, such a contradiction was absent in the dependency on claim 3, and in this dependency, the claim lacked novelty.

  • In Q18 a statement on whether the EPO would issue a Rule 62a communication in the search phase for the set of claims, which required candidates to recognize that the two independent claims would likely contravene Rule 43(2), and a statement which required candidates to recognize that Art. 123(2) allows (that is, does not disallow) amendments by which an originally filed claim is added to the description.

  • In Q19 a statement on the allowability of an amendment which represented an (unallowed) intermediate generalization.

  • In Q20 four statements on the inventive step. However, as the question failed to indicate which one of the claims is to be considered for the inventive step, question 20 was neutralized, meaning that all candidates received the full 5 marks for this question irrespective of their answers.

Comments and questions at the Tutor Meeting

Rather than showing a presentation with all Pre-Exam questions one-by-one, Stefan Götsch went through the general questions that were submitted in writing prior to the meeting and Volker Franz commented on specific questions and statements.

Stefan Götsch indicated that the Committee and Examination Board consider statistics, blog posts and comments when deciding on a possible neutralization of a statement or question, as these may indicate possible problems and ambiguities. The information is also used in the process of the Committee.

Stefan indicated that the Committee and Examination Board strive to have the answers and results available to candidates a.s.a.p. after the exam. Previously, a pdf of the paper was only made available together with the Examiner’s Report and the result (but the candidate could take his paper copy home from the exam and use that for discussing with colleagues and tutors). Since the implementation of WISEflow, the Examiner’s Report and scores are usually completed after a week, after which they are sent to the Examination Board for review and approval and for deciding on possible neutralization, and finally the scores need to be corrected via WISEflow. Last year, there was quite a lot of delay due to collection of scores through WISEflow. As a side-effect, it is not possible to announce the date of availability of exam answers in advance.

For the closure dates of the EPO in years preceding the exam, as well as in the year of the exam, candidates are referred to the relevant notices in the OJ EPO on the EPO website, which are available online in the exam.

In 2021 and 2022, the EPO legal texts (as far as HTML or small pdfs) were available online via External Resources/ Legal Texts. PCT texts were not available online yet; candidates are recommended to check notices on EQE website to check for possible changes.

Rather than being presented with trilingual questions and statements, requiring a lot of scrolling, a tutor asked to allow candidates to select a single language. Stefan indicated that WISEflow now allows, in the left column, to choose a single language for the body of the question. In the right column, all 4 statements are presented in all 3 languages. The Committee would also prefer that a single language can be selected, but WISEflow is not able to offer that. The 2023 format will be same as in 2022. New mocks will also be in the same format.

A tutor argued that randomizing the sequence of statements within a single question in the legal part and in the claim analysis part does not provide a level playing field (whereas equal treatment is of key importance according to D 11/19), as some candidates got “easier” sequences then others. Likewise, for randomizing the sequence of questions in the legal part, where some candidates got the most difficult question of a part as the first (e.g., Q.8 of part 2, directed to PCT, first). Stefan indicated that the randomization will not be changed: the Pre-Exam 2023 will use the same scheme as Pre-Exam 2022. The Committee considers that, as legal Qs are independent, a candidate is free to take them in any order, and the same for the statements, so that in their view there is no unequal treatment.

A tutor asked how the Committee and Examination Board dealt with the technical issues that some candidates had faces with the WISEflow/Lockdown Browser during the exam. For example, when launching the flow for Part 2, an endless process saying “Initialising. Preparing the assignment” just after the ID verification via camera occurred with various candidates. Stefan indicated that the complaints filed in this respect have been dealt with by the Examination Board, and that compensation was awarded where appropriate.

This year’s pre-exam did not contain any questions on time limits, while correctly calculating time limits is a key competence of a patent attorney and while earlier exams always contained multiple time limit questions. It was asked why no such questions were asked, and similarly why there was hardly any PCT in this Pre-Exam. Stefan commented that each exam may or may not have questions to particular topics and that the Committee tries to balance them. He also indicated that he cannot comment as to which topics will be covered next year. He noted that no changes have been made yet to the rules for notification, so the EQE 2023 time-limits are based on current rules (in particular, current Rule 126(2)/127(2) EPC).

A tutor noted that no decisions in Pre-Exam 2021 appeals seem to have been published, so that none were filed, the appeals were all granted in interlocutory revision, or the appeals were withdrawn. Also, no Pre-Exam 2022 appeals have been published yet. When asked about the existence of appeals to Pre-Exam 2021 and 2022, Stefan indicated that the Committee cannot comment on appeals. Likewise, the Committee refrained from commenting on whether the 22 persons that failed did indeed sit the exam and failed, or did not show up (a no-show gets 0 marks and hence is a fail), nor as to whether any specific action was taken to support these candidates in preparing for future exams.

Volker Franz commented on specific questions and statements. He said that the Committee is always checking blogs to see where any ambiguities are apparent from candidates and others that took the exam for real or as an exercise. The blogs provide important feedback for the learning process of the Committee.

W.r.t. statement 1.3, asking whether the renewal need to be paid during a period of stay or not, a tutor submitted that it is not unambiguously clear from Rule 14 EPC nor from the Guidelines whether the renewal fees that continue to fall due during the stay also need to be paid during the stay or whether they are deferred to the date of resumption (as for Rule 142(2) EPC) so that they only need to be paid upon resumption. In this context, the tutor noted that a due date is not a period (see, e.g., J 4/91, r.3.2) while Rule 14(4) only refers to periods. Also, the tutor referred to Guidelines A-IV, 2.2.4 that provides: “Stay of proceedings implies that the legal status quo existing at the time of the suspension is maintained, i.e. neither the EPO nor the parties can validly perform any legal acts while proceedings are suspended (J 38/92)”. However, if payment would be required and non-payment would lead to a deemed withdrawal, the legal status would change, contrary to this provision. Hence, the tutor considered that one could argue the case law on Rule 142(4)Both Rule 142 (4) as well as Rule 14(4) use similar wording w.r.t. periods for renewals while both are silent on due dates. Hence, in the absence of case law on Rule 14(4), one could consider the case law on interruption under Rule 142(4) (Guidelines E-VII, 1.5; J 902/87 mutatis mutandis) to be applicable mutatis mutandis lead to the conclusion that the due dates of renewals falling due during the dead period would be deferred to the date of resumption – herein, it is noted that that case law is also based on the lack of “due date” in Rule 142 and the difference between due dates and periods.
However, some legal texts indicate that renewal fees need continue to be paid during the stay, using as an argument that any person can pay so that, in case the applicant should not pay renewal fees and thus cause the application to die, the claimant is free to pay the fee in his place and will also do so to keep the application alive. This however seems contrary Guidelines A-IV, 2.2.4 mentioned above.
It may also be noted that, when the situation arises in real life, the EPO issues a communication to the applicant on record informing him that which explicitly tells him that “renewal fees must also be paid if they fall due during the staying”. This may explain the lack of any case law on it (together with the rare occurrence of such situations).
to apply mutatis mutandis which would lead to the conclusion that the due date for new renewals would be deferred to the date of resumption, and hence result in an opposite answer. Volker responded that most candidates answered as expected. He also indicated that the paper needs to be a challenge for the candidates: a statement is sometimes close to the wording of the Article, Rule and/or Guidelines, sometimes further. Here the statement was not exactly the wording of the Rule, which may have caused doubt. He indicated that the comment will be considered in designing future questions.

Q.2 comprises, in the question body common to all statements: “For the following statements, assume that the right to be heard of all parties involved has been respected and no amendments have been filed during the opposition proceedings.” Some candidates commented on blogs that this wording does not exclude/prevent that the proprietor still files amendments in response to the objections in statements 2.1 – 2.4. However, the answers in the Examiner’s Report concludes for statement 2.1 and 2.3 that “…the opposition division may revoke…”, thus ignoring that the proprietor can still amend to overcome the new ground, after which the opposition division may not revoke but must maintain in amended form. A tutor suggested that, if that would not have been intended, the phrase above should preferably have read: “For the following statements, assume that, at the moment that the Opposition takes its decision, the right to be heard of all parties involved has been respected and no amendments have been filed during the opposition proceedings”, arguing that that would have excluded further submissions. The tutor asked whether it has been considered to neutralize statement 2.1 and 2.3 in view of the wording used. In response, Volker answered that no neutralisation was considered, as the question was sufficiently clearNote from the editor: D 2/21 addresses how Pre-Exam question needs to be interpreted by a candidate when answering the exam paper. D 2/21, referring to D 5/16, r. 33, clarifies that candidates shall not dig for exotic exceptions and candidates cannot twist the interpretation to their advantage, and that it is rather a question of the correct interpretation or general understanding, rather than looking for exceptions or far-fetched interpretations.
D 2/21, reason 5: “However, if a statement is logical and makes sense, so that, using common sense, it is clear what answer was expected, candidates cannot rely on exceptions to the rule or explore alternative interpretations with a view to showing that a different answer might also be conceivable in specific instances (see e.g. D 5/16, point 33 of the Reasons). It follows that in the case of a pre-examination, the review requested by the appellant does not concern the question of whether the evaluation of the assessment of the respective statement stricto sensu, i.e. the appellant’s assessment of the statement concerned as “True” or “False”, is correct. It is rather a question of the correct interpretation or the general understanding of the statement concerned, including the facts underlying the pre-examination question and the conclusion to be drawn therefrom as to whether the statement concerned is clearly to be assessed as “True” or “False” […]”“. Refer to the full decision for details.
.

W.r.t. statement 3.4 (reading “A board of appeal may base its decision on a fresh ground for opposition only submitted during the appeal proceedings without the consent of the patentee”, a candidate submitted that the answer shall be TRUE, as the situation is not clearly scoped and that, in particular, the statement does not indicate whether the fresh ground is raised by the (original) opponent, the board, or someone else. The candidate considered it an invitation to check whether there are situations where a fresh ground of opposition may be considered and found that an intervener may raise a fresh ground, after which the Board of appeal must decide to remit the case to the first instance opposition division (G 1/94, reason 13G 1/94, reason 13: “If a fresh ground for opposition is raised by the intervener, the case should be remitted to the first instance for further prosecution unless special reasons present themselves for doing otherwise, for example when the patentee himself does not wish the case to be remitted”). Volker responded that the question was directed to general principles for proceedings before the Board of Appeal in opposition. Considering an intervention scenario, allowing fresh grounds, was not expectedSee also footnote above on D 2/21..

For statement 10.4, the statement in the actual exam and that in the exam as initially published in the compendium, and in the initial Examiners’ Report were differentIn the real exam, statement 10.4 reads: “10.4 To transfer the status of opponent to a different person during opposition proceedings, it is sufficient to file a declaration including the names, addresses and signatures of both the original opponent and the person wishing to take over the status of opponent.”
In the exam and the Examiners’ Report as initially published in the compendium it read: “10.4 In order to request a transfer of the status of opponent to a different person during opposition proceedings, it is sufficient to file a declaration including the names, addresses and signatures of both the original opponent and the person wishing to take over the status of opponent.”
. This was due to an initial use of a wrong version in the compendium (exam and Examiner’s Report), which were updated shortly after it was recognized to be incorrect.

In statement 19.4, claim II.4 was amended to include the limitation that the alcoholic beverage “having 10 vol.% alcohol or more”. The amended claim also omitted the phrasing “in a bottle” of original claim II.2, but that deletion was not shown by strikethrough or otherwise indicated. A tutor asked whether the later omission was intentional, and whether the lack of indication of its deletion was not creating un unequal playing field between candidates that did not see the deletion (and hence no potential issue) and those that did (and thus had to also take that issue into account in considering whether the amendment was allowable). Volker responded that the presence or absence of “in a bottle” has no impact on the answer. Also, it was not considered an unfair playing field for candidates that did/ did not see it. As no minutes are made from all Committee meetings, it could not be reconstructed whether it was deleted on purpose or by accident.

Due to an error in the German version in [005] (wherein should have read Poly-Y and not Poly-X, see above), the Examination Board decided to award full marks to all candidates for questions 11 to 15. Also Q.20 was neutralized, due to an accidentally omitted sentence in the question body (indicating in respect of which claim inventive step had to be assessed). It was asked how these errors could have occurred and what measures will be taken to prevent such errors (similar errors happened in D 2022, Q.5: 2017 vs 2019 in English version; in B 2021: not all amendments marked)? Volker indicated that new checks will be added to the review process; also the actual WISEflow version will be checked more carefully.

A tutor asked whether it was considered to inform all candidates during the Exam that there was an error in the paper and how it had to be corrected, so that the exam could have continued without the need for any neutralization? Volker mentioned that, when the exam was on paper in exam centres, candidates were informed during the exam when an error was found. However, in view of the short 70-minute durations of each part, there may not be enough time to note that there is an error and also to inform all candidates while having an equal playing field for all.

Nicolas Favre (Examination Board) indicated that the Examination Board, software/system providers and all others involved are still learning and improving.

9) Concluding remarks

The annual meeting of EQE tutors and members of the EQE Committees and of the Examination Board took place on 8 and 9 November 2022. The meeting took place by videoconference.

With this report, tutors summarize the papers, including their online format, and provide information of the points discussed at the meeting so that candidates and other tutors can also find this information. In addition, we hope that our summaries and comments can assist when reading and interpreting the official Examiners’ ReportsCandidates are reminded that “The purpose of the examiner’s report is to enable candidates to prepare for future examinations (cf. Art 6(6) of the Regulations on the European qualifying examination for professional representatives).” of the EQE 2022 papers.



    Comments